LSAT PrepTest 33 Section 1 Question 20 Explanation | Logical Reasoning

I didn't write the following blog post. It was already on the blog when I took over the URL. The following blog post may contain mistake...

I didn't write the following blog post. It was already on the blog when I took over the URL. The following blog post may contain mistakes. -Steve*** This Logical Reasoning question is from the December 2000 LSAT.The stimulus describes the predominant theory and presents an argument against it. 4 of the answer choices weaken the argument. The correct choice doesn't weaken it. It may strengthen it

Hot in Week

Popular

Archive

item